Difference between revisions of "2006 AMC 10A Problems/Problem 1"

m (fixed typo in link)
m (Fixed LaTeX issues)
Line 1: Line 1:
 
== Problem ==
 
== Problem ==
Sandwiches at Joe's Fast Food cost <math>3 each and sodas cost </math>2 each.  How many dollars will it cost to purchase 5 sandwiches and 8 sodas?
+
Sandwiches at Joe's Fast Food cost &#36;3 each and sodas cost &#36;2 each.  How many dollars will it cost to purchase 5 sandwiches and 8 sodas?
  
 
<math> \mathrm{(A) \ } 31\qquad \mathrm{(B) \ } 32\qquad \mathrm{(C) \ } 33\qquad \mathrm{(D) \ } 34\qquad \mathrm{(E) \ } 35 </math>
 
<math> \mathrm{(A) \ } 31\qquad \mathrm{(B) \ } 32\qquad \mathrm{(C) \ } 33\qquad \mathrm{(D) \ } 34\qquad \mathrm{(E) \ } 35 </math>

Revision as of 14:05, 4 November 2006

Problem

Sandwiches at Joe's Fast Food cost $3 each and sodas cost $2 each. How many dollars will it cost to purchase 5 sandwiches and 8 sodas?

$\mathrm{(A) \ } 31\qquad \mathrm{(B) \ } 32\qquad \mathrm{(C) \ } 33\qquad \mathrm{(D) \ } 34\qquad \mathrm{(E) \ } 35$

Solution

Our answer is simply $(3\cdot 5)+(2\cdot 8)\Longrightarrow 15+16=31,(A)$

See Also